Difference between revisions of "2008 AMC 12A Problems/Problem 6"

m
m (Problem)
Line 1: Line 1:
 
{{duplicate|[[2008 AMC 12A Problems|2008 AMC 12A #6]] and [[2008 AMC 10A Problems/Problem 8|2008 AMC 10A #8]]}}
 
{{duplicate|[[2008 AMC 12A Problems|2008 AMC 12A #6]] and [[2008 AMC 10A Problems/Problem 8|2008 AMC 10A #8]]}}
 
==Problem ==
 
==Problem ==
Heather compares the price of a new computer at two different stores. Store <math>A</math> offers <math>15\%</math> off the sticker price followed by a <math>\$</math>90<math> rebate, and store </math>B<math> offers </math>25\%<math> off the same sticker price with no rebate. Heather saves </math>\$<math>15</math> by buying the computer at store <math>A</math> instead of store <math>B</math>. What is the sticker price of the computer, in dollars?
+
Heather compares the price of a new computer at two different stores. Store <math>A</math> offers <math>15\%</math> off the sticker price followed by a <math>\$90</math> rebate, and store <math>B</math> offers <math>25\%</math> off the same sticker price with no rebate. Heather saves <math>\$</math>15<math> by buying the computer at store </math>A<math> instead of store </math>B<math>. What is the sticker price of the computer, in dollars?
  
<math>\mathrm{(A)}\ 750\qquad\mathrm{(B)}\ 900\qquad\mathrm{(C)}\ 1000\qquad\mathrm{(D)}\ 1050\qquad\mathrm{(E)}\ 1500</math>
+
</math>\mathrm{(A)}\ 750\qquad\mathrm{(B)}\ 900\qquad\mathrm{(C)}\ 1000\qquad\mathrm{(D)}\ 1050\qquad\mathrm{(E)}\ 1500$
  
 
==Solution==
 
==Solution==

Revision as of 12:12, 17 August 2020

The following problem is from both the 2008 AMC 12A #6 and 2008 AMC 10A #8, so both problems redirect to this page.

Problem

Heather compares the price of a new computer at two different stores. Store $A$ offers $15\%$ off the sticker price followed by a $$90$ rebate, and store $B$ offers $25\%$ off the same sticker price with no rebate. Heather saves $$15$by buying the computer at store$A$instead of store$B$. What is the sticker price of the computer, in dollars?$\mathrm{(A)}\ 750\qquad\mathrm{(B)}\ 900\qquad\mathrm{(C)}\ 1000\qquad\mathrm{(D)}\ 1050\qquad\mathrm{(E)}\ 1500$

Solution

Solution 1

Let the sticker price be $x$.

The price of the computer is $0.85x-90$ at store $A$, and $0.75x$ at store $B$.

Heather saves $$15$at store$A$, so$0.85x-90+15=0.75x$.

Solving, we find$ (Error compiling LaTeX. Unknown error_msg)x=750$, and the thus answer is$\mathrm{(A)}$.

===Solution 2=== The$ (Error compiling LaTeX. Unknown error_msg)\textdollar 90$in store$A$is$\textdollar 15$better than the additional$10\%$off at store$B$.

Thus the$ (Error compiling LaTeX. Unknown error_msg)10\%$off is equal to$\textdollar 90$-$\textdollar 15$$ (Error compiling LaTeX. Unknown error_msg)=$$ (Error compiling LaTeX. Unknown error_msg)\textdollar 75$, and therefore the sticker price is$\textdollar 750$.

See Also

2008 AMC 12A (ProblemsAnswer KeyResources)
Preceded by
Problem 5
Followed by
Problem 7
1 2 3 4 5 6 7 8 9 10 11 12 13 14 15 16 17 18 19 20 21 22 23 24 25
All AMC 12 Problems and Solutions
2008 AMC 10A (ProblemsAnswer KeyResources)
Preceded by
Problem 7
Followed by
Problem 9
1 2 3 4 5 6 7 8 9 10 11 12 13 14 15 16 17 18 19 20 21 22 23 24 25
All AMC 10 Problems and Solutions

The problems on this page are copyrighted by the Mathematical Association of America's American Mathematics Competitions. AMC logo.png